Là các mạch số học yếu hơn boolean?


12

Hãy biểu thị kích thước tối thiểu của một (không-monotone) số học ( + , × , - ) mạch tính toán một cho đa tuyến đa thức f ( x 1 , ... , x n ) = Σ e E cMột(f)(+,×,-) B ( f ) biểu thị kích thước tối thiểu của một (không-monotone) boolean ( , , ¬ ) mạch tính toánphiên bản boolean f b của f xác định bởi: f b ( x 1 , ... , x n ) = e E i : e i0 x i

f(x1,Giáo dục,xn)= =ΣeEceΠTôi= =1nxTôieTôi,
B(f)(,,¬) fbf
fb(x1,Giáo dục,xn)= =eE Tôi:eTôi0xTôi.
Các đa thức biết B ( f ) nhỏ hơn A ( f ) không? fB(f)A(f)

Nếu chúng ta xem xét các phiên bản đơn điệu của mạch - không có cổng Minus và không Không ( ¬ ) - thì B ( f ) thậm chí có thể nhỏ hơn theo cấp số nhân so với A ( f ) : ví dụ: đa thức đường đi ngắn nhất f trên K n ; sau đó B ( f ) = O ( n 3 )A ( f ) = 2 Ω ( n()(¬)B(f)A(f)fKnB(f)=O(n3)A(f)=2Ω(n)A(f)


LƯU Ý (15.03.2016) Trong câu hỏi của tôi, tôi không chỉ định các hệ số được phép như thế nào . Igor Sergeev nhớ tôi rằng, ví dụ, sau đây (đơn biến) đa thức f ( z ) = Σ m j = 1 2 2 j m z jMột ( f ) = Ω ( m 1 / 2 ) (Strassen và nhân dân của mình nhóm). Nhưng B ( f ) = 0 cho đa thức này, vì f b (cef(z)=j=1m22jmzjA(f)=Ω(m1/2)B(f)=0 . Chúng ta có thể có được fron f mộtđa biếnđa thức f ' ( x 1 , ... , x n ) của n = log m biến sử dụng sử dụng thay Kronecker. Liên kết với mọi số mũ j một đơn thức X j = i : a i = 1 x i , trong đó ( a 1 , Lỗi , a n )fb(z)=zff(x1,,xn)n=logmjXj=i:ai=1xi(một1,Giáo dục,mộtn) là các hệ số 0-1 của biểu diễn nhị phân của j. Sau đó, mong muốn đa thức là , và chúng tôi có mà A ( f ' ) + n Một ( f ) = Ω ( m 1 / 2 ) = 2 Ω ( n ) . Nhưng phiên bản boolean của f ' chỉ là một HOẶC biến, do đóf'= =Σj= =1mcjXj
Một(f')+nMột(f)= =Ω(m1/2)= =2Ω(n).
f' và chúng ta có một khoảng cách theo cấp số nhân. Do đó, nếu cường độ của các hệ số có thể là ba số mũ trong số n của các biến thì khoảng cách A ( f ) / B ( f ) có thểđược hiển thị theo cấp số nhân. (Trên thực tế, không phải là độ lớn - hơn nữa là sự phụ thuộc đại số của các hệ số.) Đây là lý do tại sao vấn đề thực sự với A ( f ) là trường hợp củacáchệ sốnhỏ(lý tưởng, chỉ 0-1). Nhưng trong trường hợp này, như Joshua nhớ lại, giới hạn dưới A ( f )B(f')n-1nMột(f)/B(f) Một(f)Một(f)= =Ω(nđăng nhậpn) của Strassen và Baur (với các hệ số 0-1) vẫn là tốt nhất những gì chúng ta có ngày hôm nay.

Câu trả lời:


9

VP0VNP0 ). Lưu ý rằng phiên bản Boolean vĩnh viễn chỉ là để quyết định xem một đồ thị lưỡng cực nhất định có khớp hoàn hảo hay không, có mạch đa kích thước.

Ω(nđăng nhậpn)ΣTôi= =1nxTôinΩ(nđăng nhậpn)x1x2xn


Xin chào Joshua: bạn đúng, vĩnh viễn là một ví dụ (mặc dù có điều kiện)! Chà, chúng tôi không biết bất kỳ ràng buộc nào dưới A (f) vĩnh viễn. Nhưng nếu các phiên bản VP và VNP không có hằng số khác nhau, thì chúng ta biết phân tách B (f) so với A (f) mà không biết ràng buộc (thực tế).
Stasys

2
Ω(nđăng nhậpn)

1
tại Joshua: đúng, điểm tốt một lần nữa. Nếu f là tổng lũy ​​thừa thứ n của tất cả n biến đơn thì B (f) nhiều nhất là n và Baur-Strassen cho thấy A (f) ít nhất bằng khoảng n lần logarit của n. Đây là cái được biết đến nhiều nhất với A (f). Vì vậy, khoảng cách rõ ràng lớn nhất được biết cho câu hỏi của tôi thực sự chỉ là logarit. (Một câu hỏi sang một bên: bạn có biết tại sao @ của tôi luôn biến mất trong các bình luận không?)
Stasys

@Stasys: Ví dụ hay. (Re: sang một bên. Tôi không nghĩ. Tôi nghĩ hệ thống sẽ tự động suy luận xem ai là người "at-ed" và nếu bạn đang gửi tin nhắn cho "người mặc định", thì nó sẽ xóa nó. Tôi nghĩ .)
Joshua Grochow 14/03/2016

Đúng. Tác giả của một bài đăng luôn được thông báo về các bình luận mới, vì vậy hệ thống sẽ loại bỏ thông báo @ rõ ràng là dư thừa.
Emil Jeřábek hỗ trợ Monica
Khi sử dụng trang web của chúng tôi, bạn xác nhận rằng bạn đã đọc và hiểu Chính sách cookieChính sách bảo mật của chúng tôi.
Licensed under cc by-sa 3.0 with attribution required.